You are on page 1of 12

Tutorial-1

Problem 1-Linear Spring Mass System and Frictionless

1 1
Lagrangian :L = Ke − V = mẋ2 − kx2
  2 2
d ∂L ∂L
Lagrang’s Equation : − =0
dt ∂ ẋ ∂x
 
∂L d ∂L ∂L
Do the derivatives : = mẋ; = mẍ; = −kx
∂ ẋ dt ∂ ẋ ∂x
 
d ∂L ∂L
Combine all together : − = mẍ + kx = 0
dt ∂ ẋ ∂x

Problem 2-Mass spring Damper

1
1
Ke = M ẋ2
2
1
V = Kx2 + M g(h + x)
2
1 1
L =Ke − V = M ẋ2 − Kx2 − M g(h + x)
2 2
1
P = B ẋ2
2
We have the generalize d coordinate q = x , and thus with the applied force
Q = f , we write the lagrange equation:

 
d ∂L ∂L ∂P
f= − +
dt ∂ ẋ ∂x ∂ ẋ
  
d ∂ 1 1
= M ẋ2 − Kx2 − M g(h + x)
dt ∂ ẋ 2 2
   
∂ 1 1 ∂ 1
− M ẋ2 − Kx2 − M g(h + x) + B ẋ2
∂x 2 2 ∂ ẋ 2
d
= (M ẋ2) − (−Kx − M g) + (B ẋ)
dt
=M ẍ + Kx + M g + B ẋ

Problem 3-RLC circuit

2
1 1 2 1
Ke = Lq̇ 2; V = q ; P = Rq̇ 2
2 2C 2
1 1 2
L =Ke − V = Lq̇ 2 − q
2 2C
We have the generalize d coordinate q , and with the applied force Q = u , we
have:
 
d ∂L ∂L ∂P
u= − +
dt ∂ q̇ ∂q ∂ q̇
       
d ∂ 1 2 1 2 ∂ 1 2 1 2 ∂ 1 2
= Lq̇ − q − Lq̇ − q + Rq̇
dt ∂ q̇ 2 2C ∂q 2 2C ∂ q̇ 2
d Q Q di
= (Lq̇) + + Rq̇ = Lq̈ + + Rq̇ = L + vc + Ri
dt C C dt
i = q̇ and q = Cvc for a capacitor. This is just KVL equation

Problem 4-Electromechanical System: Capacitor Microphone

This system has two degrees of freedom (electrical and mechanical :charge q
and displacement x from equilibrium)

3
1 1 1 2 1
Ke = Lq̇ 2 + mẋ2; V = q + Kx2
2 2 2C 2
where C = x0A−x ( is the dielectric constant of the air (F/m), A is the area of
the plate, x0 − x is the plate separation)

1 1 1 1
V = (x0 − x)q 2 + Kx2; P = Rq̇ 2 + B ẋ2
2A 2 2 2
1 1 1 1
L = Lq̇ 2 + mẋ2 − (x0 − x)q 2 − Kx2
2 2 2A 2
2
∂L ∂L q ∂P
=mẋ; = − Kx; = B ẋ
∂ ẋ ∂x 2A ∂ ẋ
∂L ∂L (x0 − x)q ∂P
=Lq̇; = ; = Rq̇
∂ q̇ ∂q A ∂ q̇
Then we obtain two lagrange equations
q2
mẍ + B ẋ + Kx − =f
2A
1
Lq̈ + Rq̇ + (x0 − x) = v
A
. Problem 5-Robotic Example

4
 
θ
q=
r
Generalized coordinates (θ angular position; r radial length; both vary)
 
τ
Q=
f
Applicable forces to each component; τ is the torque; f is the force
1 1
J = mr2; Ke = J θ̇2 + mṙ2; V = mgr sin(θ)
2 2
The power dissipation:P = 21 B1θ̇2 + 21 B2ṙ2
1 1
L = Ke − V = J θ̇2 + mṙ2 − mgr sin(θ)
2 2

∂L
 
   2 
∂L  = J θ̇ = mr θ̇ ;
 ∂ θ̇ 
=
∂ q̇ ∂L
  mṙ mṙ
 ∂ ṙ 
∂L
 
∂L  ∂θ  −mgr cos(θ)
=
 ∂L  = mrθ̇2 − mg sin(θ) ;

∂q
 ∂r 
∂L
 
∂P  ∂ θ̇ 
 B1θ̇
= =
∂ q̇ ∂L
  B2ṙ
∂ ṙ

5
The Langrange equation becomes
 
d ∂L ∂L ∂P
Q= − +
dt ∂ q̇ ∂q ∂ q̇
 2     
mr θ̈ + 2mrṙθ̇ −mgr cos(θ) B1θ̇
Q= − +
mr̈ mrθ̇2 − mg sin(θ) B2ṙ
 2         
mr 0 θ̈ B1 2mrθ̇ θ̇ mgr cos(θ) τ
+ + =
0 m r̈ −mrθ̇ B2 ṙ mg sin(θ) f

M (q)q̈ + V (q, q̇) + G(q) = Q


M (q) is the inertia matrix; V (q, q̇) is the Coriolis/certripetal vector; G(q) is
the gravity vector; Q is the input vector

Problem 6-Two Mesh Electric Circuit

Assume q1 and q2 as the independent generalizated coordinates, where q1 is


the electric charge in the first loop and q2 is the electric charge in the second
loop.
The generalized force applied to the system is denoted as Q1

6
We should know that: i1 = q̇1; i2 = q̇2; q1 = is1 ; q2 = is2 ; Q1 = U0(t)
The total magnetic energy (kinetic energy) is:
1 1 1
Ke = L1q̇1 + L12(q̇1 − q̇2)2 + L2q̇22
2 2 2
∂Ke ∂Ke
= 0; = (L1 + L12)q̇1 − L12q̇2
∂q1 ∂ q̇1
∂Ke ∂Ke
= 0; = (L2 + L12)q̇2 − L12q̇1
∂q2 ∂ q̇2
Use the equation for the total electric energy (potential energy)
1 q12 1 q22 ∂V q1 ∂V q2
V = + ; = and =
2 C1 2 C2 ∂q1 C1 ∂q2 C2
1 1 ∂P
The total heat energy dissipated: P = R1q̇12 + R2q̇22; = R1q̇1 and
2 2 ∂ q̇1
∂P
= R2q̇2
∂ q̇2
 
d ∂Ke ∂Ke ∂P ∂V
− + + = Q1
dt ∂ q̇1 ∂q1 ∂ q̇1 ∂q1
 
d ∂Ke ∂Ke ∂P ∂V
− + + =0
dt ∂ q̇2 ∂q2 ∂ q̇2 ∂q2
q1
(L1 + L12)q̈1 − L12q̈2 + R1q̇1 + = U0
C1
q2
− L12q̈1 + (L2 + L12)q̈2 + R2q̇2 + =0
 C 2 
1 q1
q̈1 = − − R1q̇1 + L12q̈2 + U0
(L1 + L12) C1
 
1 q2
q̈2 = − − R2q̇2 + L12q̈1
(L2 + L12) C2

7
Problem 7

Use q1 and q2 as the independent generalized coordinates:


ia = q̇1; iL = q̇2; Ua(t) = Q1
 
1 2 ∂Ke ∂Ke d ∂Ke
Ke = Lq̇2 ; = 0; = 0; =0
2 ∂q1 ∂ q̇1 dt ∂ q̇1
 
∂Ke ∂Ke d ∂Ke
= 0; = Lq̇2; = Lq̈2
∂q2 ∂ q̇2 dt ∂ q̇2
1 (q1 − q2)2
The total potential energy is: V =
2 C
∂V q1 − q2 ∂V −q1 + q2
= and =
∂q1 C1 ∂q2 C2
1 2 1 ∂P
The total heat energy dissipated: P = Rq̇1 + RLq̇22; = Rq̇1 and
2 2 ∂ q̇1
∂P
= RLq̇2
∂ q̇2

8
 
d ∂Ke ∂Ke ∂P ∂V
− + + = Q1
dt ∂ q̇1 ∂q1 ∂ q̇1 ∂q1
 
d ∂Ke ∂Ke ∂P ∂V
− + + =0
dt ∂ q̇2 ∂q2 ∂ q̇2 ∂q2
q1 − q2 −q1 + q2
Rq̇1 + = Ua ; RLq̇2 + =0
C
 1  C
 2 
1 −q1 + q2 1 q1 − q2
q̈1 = + Ua ; q̈2 = − RLq̇2
R C L C
By using Kirchhoff’s law, we get
 
dUc 1 Uc Ua(t) diL 1
= − − iL + ; = (Uc − RLiL)
dt C R R dt L

Problem 8- Directly-Driven servo-system

9
Te: electromagnetic torque
TL: Load torque
is ir
q1 = ; q2 = ; q3 = θr ;
S S
q̇1 =is; q̇2 = ir ; q3 = ωr ;
Q1 =us; Q2 = ur ; Q3 = −TL
The Lagrange equations are expressed in terms of each independent coordinate

 
d ∂Ke ∂Ke ∂P ∂V
− + + = Q1
dt ∂ q̇1 ∂q1 ∂ q̇1 ∂q1
 
d ∂Ke ∂Ke ∂P ∂V
− + + = Q2
dt ∂ q̇2 ∂q2 ∂ q̇2 ∂q2
 
d ∂Ke ∂Ke ∂P ∂V
− + + = Q3
dt ∂ q̇3 ∂q3 ∂ q̇3 ∂q3

The total kinetic energy in the sum of the total electrical (magnetic) and me-
chanical (moment of inertia) energies
1 1
Kee = Lsq̇12 + Lsr q̇1q̇2 + Lr q̇22(Electrical)
2 2
1
Kem = J q̇32(Mechanical)
2
1 1 1
Ke =Kee + Kem = Lsq̇12 + Lsr q̇1q̇2 + Lr q̇22 + J q̇32
2 2 2

10
Mutual inductance:
NsNr NsNr
Lsr (θr ) = ; LM = Lsrmax =
Rm(θr ) Rm(900)
Lsr (θr ) =LM cos(θr ) = LM cos(q3)(LM is magnetizing reluctance)
1 1 1
Ke = Lsq̇12 + LM q̇1q̇2 cos(q3) + Lr q̇22 + J q̇32
2 2 2
The following partial derivatives result:
∂Ke ∂Ke
= 0; = Lsq̇1 + LM q̇2 cos(q3)
∂q1 ∂ q̇1
∂Ke ∂Ke
= 0; = Lr q̇2 + LM q̇1 cos(q3)
∂q2 ∂ q̇2
∂Ke ∂Ke
= −LM q̇1q̇2 sin(q3); = J q̇3
∂q3 ∂ q̇3
We have only a mechanical potential energy:Spring with a constant ks
The potential energy of Spring with a constant ks:V = 12 ksq32
∂V ∂V ∂V
= 0; = 0; = ksq3
∂q1 ∂q2 ∂q3
The total heat energy dissipation is expressed as : P = PE + PM
1 1 1
PE = Rsq̇12 + Rr q̇22; PM = Bmq̇32
2 2 2

1 1 1
P = Rsq̇12 + Rr q̇22 + Bmq̇32
2 2 2
∂P ∂P ∂P
=Rsq̇1; = Rr q̇2; and = Bmq̇3
∂ q̇1 ∂ q̇2 ∂ q̇3
Substituting the original values, we have three differential equations for servo-

11
system
dis dir dθr
Ls + LM cos(θr ) − LM ir sin(θr ) + Rsis = us
dt dt dt
dir dis dθr
Lr + LM cos(θr ) − LM is sin(θr ) + Rr ir = ur
dt dt dt
d2θr dθr
J 2 + LM isir sin(θr ) + Bm + ksθr = −TL
dt dt
The last equation should be written in terms of rotor angular velocity ( dθdtr = ω).
Also, using stator current and rotor current, angular velocity, and position as
sate variables
dis 1 1 2
= (−R L
s r si − L isωr sin(2θr ) + Rr LM ir cos(θr )
dt LsLr − L2M cos2(θr ) 2 M
+ Lr LM ωr sin(θr ) + Lr us − LM cos(θr )ur )
dir 1 1
= (−R s L M i s − LsLM isωr sin(θr ) − Rr Lsir
dt LsLr − L2M cos2(θr ) 2
1
− L2M ir ωr sin(2θ) − LM cos(θr )us + Lsur )
2
dωr 1
= (−LM isir sin(θr ) − Bmωr − ksθr − TL)
dt J
dθr
=ωr
dt
Considering the third equation : dω 1
dt = J (−LM is ir sin(θr ) − Bm ωr − ks θr − TL )
r

We can obtain the expression for the electromagnetic torque Te developed:

Te = −LM isir sin(θr )

12

You might also like